Solution to Exercise of Polynomial Methods in Combinatorics

Solution to Exercise of
Polynomial Methods in Combinatorics

Yuda Chen
(Date: August 24, 2025, only contains chapter 2,3,6,7,8,10)
The author would like to expressed his gratitude to Prof. Shaoming Guo, who held a wonderful seminar in 2025 Spring semester.
Solution 2.1.

We can show the general case directly.

Claim. Given any N k-planes in n and there exists a non-zero polynomial of degree N1nk that vanishes on all the k-planes.

Notice that, if a polynomial with degree of D vanishes at (D+kk)+1 points on a same k-plane, then it will vanish on this k-plane. Therefore, there exists a non-zero polynomial satisfying the condition when (D+nn)>N(D+kk)+N, that is DnDkN, which means DN1nk. ∎

Solution 2.2.

Since the general process of proving the lemma using the Vandermonde determinant has been given in the question, we omit the proof using this method and instead use the proof of constructing the polynomial fj(x).

Define D={|xixj|}xi,xjS,xixj and fj(x)=dD(xd), then we have fj(xi)=0 if and only if ij so that we know ES is injective. Further, ES is an isomorphism.

Regarding the implication relation between the two lemmas, if Q vanishes on every point in S, that is, Q(xi)=0 for every i, we have ES(Q)=0, which implies Q=0. ∎

Solution 2.3.

We will use induction to prove this lemma.

If n=1, we know P have at most D zero points. Now we proceed by induction on n. We can rewrite P(x1,x2,,xn) as Q(xn)=j=0DPj(x1,,xn1)xnj. And we know Z(P)=anAn{xZ(P)xn=an}=anAn{(x1,,xn1,an)(x1,,xn1)Z(Q(an))}, so we have |Z(P)|=anAn#{(x1,,xn1,an)(x1,,xn1)Z(Q(an))}NDN(n1)1=DNn1 by induction hypothesis. ∎

Solution 2.4.
11 1 This part of the proof comes from [3].

We will use induction to prove this conclusion. Suppose this conclusion is established for polynomials in n1 variables. We can rewrite P(x,xn)=j=0dPj(x)xnj, where x=(x1,,xn1). Notice that, if (x,xn)Z(P), then x,xn must satisfy at least one condition:
(1) For any i=0,,d, Pi(x)=0
(2) Fix x, then xnZ(j=0dPj(x)xnj) (Notice that j=0dPj(x)xnj0).

First, We assume E={(x,xn)x satisfies condition (1)} and F={(x,xn)xn,x satisfy condition (2)}. By inductive hypothesis, we know m(E)=0. For any fixed x, we know there only exist at most d roots of j=0dPj(x)xnj by fundamental theorem of algebra. Now applying Fubini’s theorem then we have m(F)=0, which leads m(Z(P))=m(EF)=0. ∎

Solution 2.5.

We can take c(d,n)=12n(d+1)nn!. If this conclusion isn’t established, assuming k=min(d,q1), we can find PPolyD(𝔽qn) that vanishes at all points in a𝔽qnΓa with D=qk+11 because

(D+nn)qk+1nn!qn2n(k+1)nn!12n(d+1)nn!qn=c(d,n)qn>|a𝔽qnΓa|

We can also assume that da,j=degQa,j won’t exceed q1 (if not, we can assume da,j as da,jda,jq1(q1) to replace da,j). Further, we have da,jk and degQP,aDkqDq1 if assuming QP,a(t)=P(Qa,1(t),,Qa,n1(t),t).

Claim. If PPolyD(𝔽qn) that vanishes at all points in a𝔽qnΓa, then we have xn|P(x1,,xn) and P(x1,,xn)xn also vanishes at all points in a𝔽qnΓa.

Because of (a,0)Γa for any a𝔽qn1, P(a,0)=0 holds for any a𝔽qn1, that is, xn|P(x1,,xn). If we assume P1(x1,,xn)=P(x1,,xn)xn, we have degQP1,aq2. Now take t=1,,q1 and we know QP1,a(t)=QP,a(t)t=0 for any a𝔽qn1. Further, we know QP1,a(0)=0 for any a𝔽qn1, that is, P1(a,0)=0, which means P(x1,,xn)xn also vanishes at all points in a𝔽qnΓa.

If we assume that P is the polynomial with the lowest degree satisfying that P vanishes at all points in a𝔽qnΓa, we have degP=0, implying P0, which causes the contradiction. ∎

Solution 2.6.

Assuming the maximum number of joints generated by L lines is JL, then we have the following claim.

Claim. There exists one line with at most nJL1n joints.

First we know we can find a non-zero polynomial P with degree at most nJL1n vanishing at every joint. (Let P be the polynomial with the lowest degree that satisfies this condition.) If every line contains over nJL1n joints, then P vanishes on every line. So we know P also vanish on every joint. Due to the minimum degree of P, we have degP=0, which leads a contradiction.

Now we can know JL(nL)nn1 from

JLJL1+nJL1nJL2+2nJL1nnLJL1n

Solution 2.7.

We assume that set X contains all the joints. For each coordinate axis j, define the indicator function fj:{0,1} as:

fj(xj)={1if xjπj(X),0otherwise.

Then we know xjfj(xj)=|πj(X)||𝔏j| and the cardinality of X can be expressed as:

|X|=(x1,x2,,xn)X1=x1,x2,,xnf1(x1)f2(x2)fn(xn).

For n non-negative functions f1,f2,,fn, Hölder’s inequality states:

x1,,xnf1(x1)f2(x2)fn(xn)j=1n(xjfj(xj)n1)1n1.

Notice that (xjfj(xj)n1)1n1=(xjπj(X)1)1n1=|𝔏j|1n1, so we have |X|j=1n|𝔏j|1n1, which finishes the proof. ∎

Solution 2.8.

It’s trivial that P vanishes at all points. If there exist a polynomial Q(x1,x2,x3) with degQ|A|, we can rewrite it as i=0|A|1Qi(x2,x3)x1i and get Qi(x2,x3)0 for any i, from Q vanishing at |A| points when x2 and x3 are fixed. And we can rewrite Qi(x2,x3) as j=0|A|iQij(x3)x2j. Similarly, we know Qij0 for any j. Then we have Q0, which leads a contradiction.

Now we know that every minimal degree polynomial Q vanishing on the grid has degree of |A|. Consider that for every fixed x2 and x3, we have aA(x1a)Q. Then Q(x1,x2,x3) must be form of R(x1,x2,x3)aA(x1a). Considering the degree, we know R(x1,x2,x3) is a constant, that is, Q is a multiple of P.

For the case of |A|=|B|=|C|, we will prove this conclusion: for every minimal degree polynomial P vanishing on the grid, P must be the form of

c1aA(x1a)+c2bB(x2b)+c3cC(x3c)
22 2 This part of the proof comes from [7].

Assuming |A|=|B|=|C|=n, we can rewrite

P(x1,x2,x3)=c1aA(x1a)+c2bB(x2b)+c3cC(x3c)+Q(x1,x2,x3)

where Q satisfying:
(1) Its degree is at most n;
(2) None of its monomials contains the nth power of a single variable;
(3) It vanishes on any combination where each of its variables is drawn from a fixed set of n distinct points.

Claim. Any polynomial Q(x1,,xm) satisfying these three properties must be the zero polynomial.

We will prove this conclusion by induction on the number of variables. It’s obvious that the conclusion establish when m=1 because we know it has degree at most n1 from condition (1) and (2). Suppose that the conclusion holds for polynomials with m1 variables. Let Q(x1,,xm) satisfy the three conditions and vanish on A1××Am where |Ak|=n. Now we can rewrite Q as k=0n1Qk(x1,,xm1)xmk, then we know Qk(x1,,xm1)0 by degxmQn1 and condition (3). Then we know that Qk vanishes on A1××Am1 and also satisfies condition (1) and (2), for any k. Now we can know Qk0 by induction hypothesis, which finishes the proof.

Finally, let m=3 and the proof of the conclusion is completed. ∎

Solution 2.9.

The conclusion of this exercise can be fully supported by the proof presented in [8]. ∎

Solution 3.1.

Firstly, we can define v,w in 𝔽q3 as vw¯, which equals to i=13viwip, so every point x on the Hermitian variety satisfies x,x=1.

Claim. For any v,wH, there exist gU(𝔽q3) satisfying gv=w.

For any vH, we can extend it to an orthonormal basis {v,e2,e3} satisfying

v,v=1,v,ei=0,ei,ej=δij

Also, we can construct another orthonormal basis {w,f2,f3} satisfying

w,w=1,w,fi=0,fi,fj=δij

Then let g(v)=w,g(e2)=f2,g(e3)=f3, it’s obvious that g keeps the inner product, which implies U(𝔽q3). Now we know this group action is transitively. ∎

Solution 3.2.

Firstly, about the number of points on hypersurface Q, we can rewrite the equation as x12+x22=1+x32+x42 and let 1+x32+x42=c. By results in [6], we know the equation x12+x22=c has q solutions for (x1,x2) if c is fixed. Allow x3 and x4 to vary, then we know the number of the solutions is q3, which means there are q3 points on Q.

Secondly, about the number of lines lying on Q, we need the following claim.

Claim. Each point of Q lies on q lines.

Consider that (x1+v1t)2+(x2+v2t)2(x3+v3t)2(x4+v4t)2=1 always holds for every t, it implies x1v1+x2v2x3v3x4v4=0 and v12+v22v32v42=0. Let Sr={(v1,v2,v3,v4)𝔽q4v12+v22=v32+v42=r} and it’s obvious that Sr contains q2 points for fixed r by [6]. Notice that x1v1+x2v2x3v3x4v4=0 are linear for (v1,v2,v3,v4), so the number of points on Sr satisfying the above linear equation is q. Allow r to vary from 0 to q1, we know the number of solutions for (v1,v2,v3,v4) of initial equations is q2. And because the above equations which contain vi are all homogeneous, there are only q2q1q different directions, that is, each point corresponds to q lines.

Because each point contributes q lines and there are q3 points on Q, each line is shared by q points. Now we know there are q3qq=q3 lines on Q.

Finally, regarding of fact that the lines through each point xQ lie in a 3 -plane, we can know every line passing x is in x1v1+x2v2x3v3x4v4=0, which is a 3 -plane, by the knowledge of linear algebra. ∎

Solution 3.3.

For the case of n=1, we can consider the linear map E:W𝔽D+1,f(f(x0),,f(xD)). If dimWD+2, then there exist linear independent functions f1,,fdimWFcn(𝔽,𝔽), and we know dimkerE+dimImE=dimW by Rank-Nullity Theorem, which leads dimkerE1. So there exists a non-zero function g=c1f1++cdimWfdimW satisfying E(g)=0, which is a contradiction.

When n2, we assume that the case of n1 is established. If regarding P(t1,,tn) as a function with respect to tn, we have

P(t1,,tn)=k=0DPk(t1,,tn1)tnk

where PkFcn(𝔽n1,𝔽) (the reason why we can make this decomposition is that {1,tn,,tnk} forms a group of basis). Notice that Pk satisfies vanishing lemma of degree D, we have

dimW(D+1)n1choices of P0+(D+1)n1choices of P1++(D+1)n1choices of PD=(D+1)n

according to the induction hypothesis. ∎

Solution 6.1.

We use the Bezout theorem in [4] to prove the following claim.

Claim. If degP,degQ<|𝔽|, then the theorem is still true when |𝔽| is finite.

Firstly, we know

ji(Y,H;Zj)degZj=degYdegH

from [4]. We consider P=0 and Q=0 as algebraic curve defined in 𝔽q¯ and here Y=V(P),H=V(Q)𝔽q¯2. So for every line in initial Z(P,Q), it’s also in the ”new Z(P,Q)”.

Let lines be L1,,LrYH and now

j=1ri(Y,H;Zj)degZjd1d2

Because Zj are lines on P=0 and Q=0, degZj=1 and i(Y,H;Zj)1 hold. So

j=1ri(Y,H;Zj)degZj=j=1ri(Y,H;Zj)r

which means

rji(Y,H;Zj)degZjd1d2

Notice that the lines in initial Z(P,Q) are all in {Li}i=1r then the claim is true. ∎

Solution 6.2.

For the first question, parameterize curve P(𝔽) as (P1(t),P2(t)) and define polynomials

f(t)=P1(t)x,g(t)=P2(t)y,

here x,y are varible. Because degP1D and degP2D, we have degtfD and degtgD. So resultant rest(f,g) is polynomial wrt x,y and let Q(x,y)=rest(f,g). When (x,y)=(P1(t),P2(t)), we know f and g have common root t, then the resultant equals to 0. So Q(P1(t),P2(t))=0 holds for every t, which means P(𝔽)Z(Q). Consider the form of resultant (a 2D×2D determinant), we know x,y only appear in the right D columns, so degQD.

For the second question, firstly parameterize H:

s(u)=u2+12u,t(u)=u212u,

and here u𝔽{±1}, then let

X(u)=P1(s(u),t(u)),Y(u)=P2(s(u),t(u)).

Let X(u)=A(u)/C(u) and Y(u)=B(u)/D(u), here A,B,C,D are polynomials and degA,degC2D, degB,degD2D.

f(u)=C(u)xA(u),g(u)=D(u)yB(u).

Now let Q(x,y)=resu(f,g). Notice that C(u) and D(u) are all like (2u)k and kD. Similar as the first proof, just consider the 4D×4D determinant, we know degQ2D because there are only 2D columns containing x,y.

For the general case, if P:H𝔽k is a polynomial map where H is a curve or (hyper)surface with dimension of r and degree of m, and every degPi is not more than D, then there exist Q such that P(H)Z(Q) and degQmDr. The details of proof can be read here: [1]. ∎

Solution 7.1.
33 3 Hint of this problem may has defect. This solution has fixed that.

Using the hint of exercise, for an integer a2, let D(a,Q) be the set of pairs (p,q) with 1p<qQ, and so that a divides both p and q. So we have |D(a,Q)|(Qa2)(Q2)a2.

Note that R(Q) can be formed by starting with all pairs (p,q) with 1p< qQ and then removing D(a,Q) for every prime a. Therefore,

|R(Q)|(Q2)a prime|D(a,Q)|(1a2, primea2)(Q2)

When Q5, we have (Q2)=Q1QQ2225Q2, which means we only need prove that 1a2, primea214. And we can use a=2a2<23 to finish this proof. When Q=2,3,4, the ratios are 14,13,516 respectively. So we have |R(Q)|110Q2. ∎

Solution 7.2.

Let S(r) be the set of fractions in S(r) whose numerator and denominator are both at least r1210. We will state that |S(r)|r2. We only need to prove that there are no more than r2 fractions whose numerator or denominator is less than r10. Notice that k=1nφ(k)n24 (just consider φ(k)k+12 for odd k), then every fraction in S(r) has a denominator less than 4r. So there are at most r104rr5 fractions with numerator greater than r10 and at most k=1r10φ(k)k=1r10k<r100 fractions with denominator greater than r10. Sum them up then we know the conclusion is true.

Let 𝔏N,r𝔏N,r be the lines with slope in S(r). Then every line with slope of pq in 𝔏N,r intersects the N×N grid GN at most 1+min(N1p,N1q)1+10NrNr12 points. But every point in GN lies in at least r2 lines of 𝔏N,r. By double counting, we know |𝔏N,r||𝔏N,r|N2Nr12r2Nr32, so the proof finishes. ∎

Solution 7.3.

Every line in 𝔽q2 can be written as ax+by+cz=0 where [x,y,z] represents point on the line and (a,b,c)(0,0,0). Then line a1x+b1y+c1z=0 intersects line a2x+b2y+c2z=0 at only [b2c1b1c2,a1c2a2c1,a2b1a1b2]. Also, for a fixed point [x,y,z] in 𝔽q2, because ba can take q+1 different values (including ), there are q+1 different lines in 𝔽q2 passing it. Then we know |Pq+1𝔏|=(|𝔏|2)(q+12) immediately. ∎

Solution 7.4.

With some simple calculation, we know LHr. For every (x0,y0) satisfying |x0|Hr1 and |y0|H, we have y=mx+(y0mx0) passing (x0,y0), for m=1,2,,r (notice that |y0mx0|2H), which means every (x0,y0) is an r-rich point. So Pr(𝔏)Hr1HL2r3. ∎

Solution 7.5.

Take 𝒮=Pr(𝔏) in Theorem 7.11 and then we have I(Pr,𝔏)S23L23+S+L. Also, we have trivial bound of I(Pr,𝔏)Sr (for every point, only consider the r lines passing through it). Now we have SrS23L23+S+L. Because S(L2)L22<L2, we have S23L23>S.

If S2L, which implies LS23L23, we have SrL, that is, SLr1.

If S2L, which implies LS23L23, we have SrS23L23, that is, SL2r3.

Then we have |Pr(𝔏)|=SLr1+L2r3, which is Theorem 7.1. ∎

Solution 7.6.

Using 𝔏 and 𝒮 we construct a graph drawn in the plane. The vertices of our graph G are the points of 𝒮. We join two vertices with an edge of G if the two points are two consecutive points of 𝒮 along a line 𝔏. We let k denote the number of points on . Notice that |I(𝒮,𝔏)|=𝔏k1 and E=𝔏max(k1,0), we have |I(𝒮,𝔏)|E+L. The crossing number of this drawing is at most (L2)L2, since each crossing of the graph G must correspond to an intersection of two lines of 𝔏. This is also a straight line drawing. So we see that L2kstr(G).

Claim. We have Emax(4L23S23,4S).

If E4S, we can apply the estimate kstr(G)164E3S2, then we get L2164E3S2, which means E4L23S23. If E<4S, the conclusion holds naturally.

Now we know I(𝒮,𝔏)E+LS23L23+S+L immediately. ∎

Solution 7.7.

Proof by Theorem 7.11: Take 𝔏=𝔏r in Theorem 7.11 and then we have I(𝒮,𝔏r)S23L23+S+L. Also, we have trivial bound of I(𝒮,𝔏r)Lr (for every line, only consider the r points lying on it). Now we have LrS23L23+S+L. Because L(S2)S22<S2, we have S23L23>L. If L2S, which implies SL23S23, we have LrS, that is, LSr1. If L2S, which implies SL23S23, we have LrL23S23, that is, LS2r3. So the conclusion is established.

Proof by Theorem 7.1: Consider the polarity transformation

f:(a,b)ax+by=1,ax+by+c=0(ac,bc)

where c0, assuming 𝒮,𝔏 both don’t contain or pass origin point, we have f(𝒮) is a set of lines and f(𝔏) is a set of points. And for every point Q, we have f(Q) passing Q. For every line , we have f() lying on . By Theorem 7.1, we know |Pr(𝔏)|Lr1+L2r3. We get the conclusion since

|𝔏r|=|f(Pr(f(𝒮)))|=|Pr(f(𝒮))||f(𝒮)|r1+|f(𝒮)|2r3=Sr1+S2r3

Solution 7.8.

Let S=[xi,yi,zi]i=1n, there must exist α such that xi+αyi+α2zi0 for i=1,2,,n (just let α not equal to the root of xi+αyi+α2zi=0 for every i). So the projective transformation

p:(xyz)(1000101αα2)(xyz)

can map S to a set in 2. Because projective transformation don’t change the number of lines, points and incidences, directly, we know that Szemeredi-Trotter Theorem of 2 version holds. Because every line in 2 corresponds to unique point lying on it (vice versa), let this map be f, we have

Imax(𝒮,𝔏)=Imax(f1(𝔏),f(𝒮))=Imax(p(f1(𝔏)),p(f(𝒮)))

If we assume Imax(𝔏,𝒮)=g(S,L), then we have Imax(p(f1(𝔏)),p(f(𝒮)))=g(L,S), which means Imax(𝔏,𝒮) is symmetric for S and L. ∎

Solution 7.9.

Firstly, it’s trivial that there doesn’t exist a component with only one side. If there exist a component K with two sides, assuming they’re 1,2 and P=12, then consider 3 that intersects 1 and 2 at points other than P. Such a line 3 must exist, because if not, then every line passes P. Notice that 3 intersects area K, we know K isn’t connected, which leads a contradiction.

Let Vk be the number of vertices on exactly k lines and Fk be the number of faces with exactly k sides. Notice that 2E=k=3kFk3F and E=k=2kVk, if V2=0, we have E3Vk. Now

VE+FVE+23E=V13EV13(3V)=0

but we know VE+F=1>0 by Euler’s formula. So that leads a contradiction, which means V21.

Using the map f in the previous question, we can get the claim quickly:

Claim. Suppose that 𝒮 is a finite set of points in 2, not all lying on one line. Then, there is a line that contains exactly two points of 𝒮.

Just use the conclusion above for f1(𝒮), we know there exist a point v on exactly two lines v1 and v2. Then line f(v) contains exact two points f(v1) and f(v2). The claim is true for 2, so the claim is of course true for 2. ∎

Solution 7.10.

Because the vi are chosen generically, the values of vi+vj(ij) and values of vi+vj+vk(ij,jk,ki) are all distinct. So the number of points in X is N=(m2)m2. Let Cij denote the unit circle centered at vi+vj. Then every point with form vi+vj+vk is on circle Cij,CCjk,CkiΓ, which implies vi+vj+vk is 3-rich. Thus the number of 3-rich point of Γ is at least (m3)m3N32. ∎

Solution 7.11.

For every point (x0,y0) such that 1x0A and y0A, it lies on y=(x0a)x(a+y0) for every aA. So (x0,y0) is a r -rich point and Pr(𝔏)|A|2. By Theorem 7.1, we know

|A|2 |A+A||AA||A|1+|A+A|2|AA|2|A|3
=|A+A||AA||A|2|A|3+|A+A|2|AA|2|A|3
2|A+A|2|AA|2|A|3

then we have |A+A||AA||A|52, which means max(|A+A|,|AA|)|A|54.

Regarding of the case max(|AA|,|A/A|), let 𝔏 be the set of lines y=nxc with nA/A and cAA. For every point (x0,y0) such that x0A and y0A, it lies on y=ax0x(ay0) for every aA. So (x0,y0) is a r -rich point and Pr(𝔏)|A|2. By Theorem 7.1, we know

|A|2 |AA||A/A||A|1+|AA|2|A/A|2|A|3
=|AA||A/A||A|2|A|3+|AA|2|A/A|2|A|3
2|AA|2|A/A|2|A|3

then we have |AA||A/A||A|52, which means max(|AA|,|A/A|)|A|54. ∎

Solution 7.12.

Consider the simple graph H corresponding to G, which has the same vertices as G. Every edge of G (regardless of the number of repetitions) corresponds to a single edge in H. Let EH be the number of edges in H. We have EMEH. Thus there is EHEM4MVM=4V. Using Theorem 7.6 and we know k(H)164EH3V2(EM)3V2. Notice that k(G)k(H), we get k(G)164E3M3V2. ∎

Solution 7.13.

Let P denote the set of N points. Assuming |d(P)|=tN, let P be vertices of G and we consider the circles centered at points in P with radii in d(P). The number of circles C=Nt. Let the edges of G be the arcs of circles between consecutive points, for the circles containing more than one point. We don’t construct an edge for the circles containing only one point to avoid loops.

Notice that, for every circle with more than one point lying on it, the number of arcs equals to the points. Let U be the circles with only one point. Then E=N(N1)|U|N(N1)C=N(N1t). Because there are up to 99 points on the perpendicular bisector of every two points, the number of parallel edges between two points is at most 198, which means Mult(G)198.

Firstly, consider the case of E4MV=792N, we can use Proposition 7.13 and get k(G)164E31983N2. In addition, there is k(G)2(C2)(Nt)2. Therefore, we have

(Nt)2k(G)164E31983N2164N3(N1t)31983N2

Assume u=tN<110 and N2, then

u2164(11Nu)31983(12u)364198323

which means

1641983u3+(111281983)u2+12561983u15121983

Thus, we have 2u15121983, that is, t17948689408N. We call c as 17948689408.

For the case of E<792N, we have 792N>N(N1t), which is equivalent to t>N793. When N794, we know t>N793>cN. When N793, because t1, tcN is established. All in all, we have t17948689408N. ∎

Solution 7.14.

Let S be vertices of G and we consider the unit circles centered at points in S. Let the edges of G be the arcs of circles between consecutive points, for the circles containing more than one point. Let mc={pS|pc|=1} denote the number of points on the unit circle centered at cS and D denote the number of unit distances. Then we have E=cS,mc2mc,2D=cSmc and Mult(G)4. Further, we have

D=cSmc2=E+|{cSmc=1}|2

Because two unit circles intersect in at most two points, we have k(G)2(N2)N2. When E16N, we know E34096N2k(G)N2, which means E16N43. So there is D16N43+N2172N43. When E<16N, we also have DE+N2172N172N43. Thus the conclusion is established. ∎

Solution 7.15.

Use the following method to construct the graph G: Let P=Pr(Γ) be vertices of G. For every curve γΓ, we can consider Pγ=γP, the set of points on γ. Let the edges of G be the arcs of curves between consecutive points, for the every curve γ with |Pγ|2.

Assuming that eγ is the number of edges in G on γ and mγ=|Pγ|, for mγ2, we know eγ=mγ if γ closed and eγ=mγ1 if γ open. Because any two points lie in at most s curves of Γ, we get Mult(G)2s.

Notice that every point pP is r -rich, we have γΓmγ=pP{γ:pγ}rV. And notice that max(mγ1,0)eγmγ, we have

E=γΓeγγΓmax(mγ1,0)γΓ(mγ1)=γΓmγLrVL

Because any two curves of Γ intersect in at most s points, we know k(G)s(L2)sL22.

If E4MV=8sV, we get E3512V2s3k(G)sL22, which means E283s43L23V23. By ErVL, we know rVL283s43L23V23. Let K denote 283s43. For the case of rV2L, we know rV2rVLKL23V23, that is,

V(2KL23r)3=8K3L2r3=2048s4L2r3

For the case of rV<2L, we have V<2Lr1.

If E<8sV, from ErVL, we know V(r8s)L. For the case of r16s, we have VLr8sLr2=2Lr1. For the case of r<16s, by Vs(L2)sL22, we have

VsL22=sL22r3r3sL22r3(16s)3=256s4L2r3

In summary, we get |Pr(Γ)|C(s)(L2r3+Lr1). ∎

Solution 7.16.
44 4 The condition rN13 in this problem should be rN13, or we have Lr1=Nr1<N and L2r3=N2r3<N.

Consider the set of parabolas Γ={x2+ax+ba=0,1,,k1,b=0,1,,k21} with |Γ|=N=k3 and set of points S={(m,n)0mk+1,n=0,1,,k21m(k1)}{(m,n)k1m1,n=m(k1),m(k1)+1,,k21} with |S|=k3+k2+k+2N and |S|N. Because every (m,n)S satisfies n=m2+am+b for (a,b)=(0,nm2),(1,nm2m),,(k1,nm2(k1)m), we know every point in S is r -rich point, with r=kN13. So the conclusion is established. ∎

Solution 7.17.

Firstly, we will show this conclusion.

Claim. Curve γ0 intersects γ0+v in at most 2 points, assuming v0.

WLOG, we can assume that v=(0,m) with m>0. Choose a and b as the left and right endpoint of γ0 (that is, the points on γ0 with the smallest and largest x coordinate). Let fu(x)=max{y(x,y)γ0} and fl(x)=min{y(x,y)γ0}. Then we have γ0={(x,fu(x))x[a,b]}{(x,fl(x))x[a,b]} and γ0+v={(x,fu(x)+m)x[a,b]}{(x,fl(x)+m)x[a,b]}. 55 5 This part of the proof comes from [5].Since m>0, graphs of fu and fu+m are disjoint, the graphs of fl,fl+m are disjoint and the graphs of fl,fu+m are also disjoint. Only the graphs of fl+m and fu can intersect. Then, the conclusion reduces to the claim that the graph of a strictly convex function h1 can intersect the graph of a strictly concave function h2 in at most two points. Notice that h1h2 is strictly convex, we know it has at most 2 roots, so the claim is established.

If there are 3 curves γ0,γ0+t,γ0+s all passing p,q, then γ0 passes p,pt,ps and q,qt,qs. That means γ0 intersects γ0+(qp) at 3 points, which leads a contradiction. So any two points in the plane lie in at most two of the curves of Γ.

Now applying Theorem 7.16, we know |Pr(Γ)||Γ|2r3+|Γ|r1. Suppose that γ0 has diameter of d, assuming d1, we choose a 10d×10d square B such that γ0B¯. We call ΓB as {γ0+vv2,(γ0+v)B¯0} and r(p) as the number of curves in Γ passing p, assuming Γ={γ0+vv2}. Consider 𝒫=B¯2 and S=γ02 with |S|=n, we know for every point p𝒫,

r(p)=|{v2pvγ0}|=|{sγ0s=pv for some v2}|=|S|=n

which means every p is contained by n curves in ΓB. Notice that |ΓB|kd2 and |𝒫|cd2, we know

cd2|𝒫||Pn(Γ)||ΓB|2n3+|ΓB|n1=k2d4n3+kd2n1

by the inequality above. So we have ck2d2n3+kn1.

For the case of k2d2n3kn1, we know n(2k2c)13d23 because ck2d2n3+kn12k2d2n3. For the case of k2d2n3kn1, we have ck2d2n3+kn12kn1, so there is n2kc2kcd23. Anyway, the conclusion is established. ∎

Solution 7.18.

We’ll prove this claim firstly.

Claim. For every G with E>22iMV satisfying that between any two vertices of G, the number of edges is either 0 or lies in [M2i+1,M2i], its crossing number k(G)2i8E3V2M1.

Consider the simple graph H corresponding to G, we have 2iEMEH2i+1EM, which implies EH>22i2iMVM=4V. Take a random subgraph GG consisting of one edge between each pair of vertices. In the induced embedding on the subgraph, each crossing occurs with probability (2i+1M)2, since it occurs if and only if both edges are in G. Now we have 𝔼(k(G))22i+2M2k(G), that is,

k(G)22i2M2𝔼(k(G))16422i+2M2EH3V22i8E3V2M1

since EG=EH>4V and EH2iEM.

66 6 This idea comes from [9].

For 0i[log2m], let Gi be the subgraph of G, satisfying that between any two vertices of Gi, the number of edges is either 0 or lies in [2i,2i+1). Define A={iEGi2i+3V} and B={0,1,,log2M}\A, from iAEGi22[log2M]+316MV, we know

iBEGi=EiAEGiE16MVE2

Notice that

k(G)i=0[log2m]k(Gi)iBk(Gi)28V2M1iB2iEGi3

by Holder inequality, we know

iB2iEGi3=iBEGi32i(iBEGi)3(iB2i2)2(E2)3

so we have k(G)211E3V2M1=12048E3V2M1. ∎

Solution 7.19.

We let kC be the number of points on circle CΓ. Let fp denote the number of circles containing point p. Now we know

E=CΓkC=pSfp

Let gp be the number of circles in Γ0\Γ, only containing p and sq be the number of circles in Γ0\Γ, centered at q. Notice pSgp=qSsqqSt=Nt, we have

qSsqqSt=Nt1100N2

So

E=pSfp=pS(N1gp)=N(N1)qSgqN(N1)1100N212N2

for N3. ∎

Solution 7.20.
77 7 Besides hint, this idea also comes from [9].

We assume tcN45, otherwise we have tN45 trivially. By Lemma 7.19, the number of edges of G with multiplicity at least M is at most C[N2M2t+NlogNt]. We take M=12Ct, then the value above changed into N212+CNlogNt. Using the bound tcN45, we know

EM EG(N212+CNlogNt)12N2(N212+CNlogNt)
=512N2CNlogNt512N2CNlogcN95=N23

when N sufficient large. Therefore we have EMN2310012CN75=100MV when N sufficient large. Now apply Theorem 7.17 on GM and we know

k(GM)c0EM3V2M1c0(N23)3N2M1=c027N4M1

Using k(GM)k(G)2(Nt)2, we know

2N2t2c027N4M1=c027N4(12Ct)1=c02712CtN4

Thus we have t(c05412C)25N45, which means tN45. That is, we have tN45. ∎

Solution 8.1.
88 8 Hint of this problem may has defect. This solution has fixed that.

For any point x=(x1,x2,x3)3, with x0,1, we define a map ρx:22 as follows. If v2, then we define ρx(v) so that (v,0), x and (ρx(v),1) are collinear. Now observe that x is r -rich if and only if |ρx(G0)G1|r. Construct a vertical line perpendicular to plane x3=0 through point x, assume intersects x3=0 and x3=1 at h0,h1 respectively. We can get xh0(v,0) is similar to xh1(ρx(v),1), which means px(v1,v2)=1x3x3v+1x3(x1,x2). Let k=1x3x3 and t=1x3, we have ρx(v)=kv+tx. We let fk(x)=|{a{1,2,,N}ka+tx{1,2,,N}}|, where N=L1420, then x is r -rich if and only if fk(x1)fk(x2)r. Let’s consider k=qdq, where q,d are both positive integers satisfying gcd(q,d)=1 and 0<d<2q. Now x3=qd,t=dq and k=1dq.

Claim. Assuming N2q, then for k=qdq, the number of x satisfying fk(x)N2q is at least Nq.

For x=1d,2d,,Nqd, we know qdq((qd)1dx+mq)+dqx{1,2,,N}, for m=0,1,,[Nq]1, where (qd)1 is the number-theoretic inverse of qd modulo q. Notice m can take [Nq]N2q values, we know the claim is established.

When r is fixed, let Q=N4r. For every (q,d) pair, which satisfies gcd(q,d)=1,0<d<2q and qQ, contribute (Nq)2 choices for (x1,x2) pairs. (Because 2rN2400, we know qQN2r, which means N2rq2q, we can use the claim above.) And for every fixed q, there are 2φ(q) choices of d, for gcd(q,d)=1 and 0<d<2q. Now the number of lines passing x is fk(x1)fk(x2)(N2q)2(N2Q)2r.

Then we know

|Pr(𝔏)| q=1Q2φ(q)N2q2=2N2q=1Qφ(q)q22N21qQ,q odd1+q2q2
N2Q(Q+2)(3Q22Q2)24N2Q412112L32r2L32r2

when Q3. Since QN4r=N216r5, we know the inequality above is true. ∎

Solution 8.2.

Step 1: For every yS(u,r)S(v,r), it satisfies i=13(uiyi)2=r and i=13(viyi)2=r. Then we know

0=i=13(uiyi)2(viyi)2=i=13(ui2vi2+2yivi2yiui)=i=13(ui2vi2)2yi(uivi)

which means i=13(uivi)yi=i=13(uivi)21(ui+vi).

Step 2: If Perp(u,v)=Perp(u,w), we know there exist c0 satisfying uivi=c(uiwi) for i=1,2,3 and i=13(uivi)21(ui+vi)=c(i=13(uiwi)21(ui+wi)).

Step 3: Notice that S(u,r)S(v,r)S(w,r)Perp(u,v),Perp(v,w) and Perp(w,u), thus S(u,r)S(v,r)S(w,r)Perp(u,v)Perp(v,w)Perp(w,u). We can know Perp(u,v)Perp(v,w)Perp(w,u) is a line from the equation of plane. We call this line as .

Step 4: From S(0,r) doesn’t contain any line, we know S(u,r) also doesn’t contain any line. Since S(u,r)S(v,r)S(w,r)S(u,r), we consider the points in |S(u,r)|. It can be realized as finding roots of a quadratic equation whose coefficients are not all 0. (Specifically, we substitute :y=at+b into the equation S(u,r) to solve for t.) Thus we know |S(u,r)|2, which leads the conclusion.

Step 5: Suppose that S(0,r) contains the line parametrized by γ(t)=mt+b, where m=(m1,m2,m3) and b=(b1,b2,b3). This is equivalent to saying that i=13(mit+bi)2r=0 for all t𝔽q3. If we expand the left-hand side as a polynomial in t, each coefficient must vanish, leading to the equations

i=13mi3=0;i=13mibi=0;i=13bi2=r

These formulas lead to a tricky way of writing r as a negative square. Since 1 is a quadratic residue, we can conclude that r is a quadratic residue.

m12r =m12(b12+b22+b32)=(m1b1)2+m12(b22+b32)
=(m2b2+m3b3)2(m22+m32)(b22+b32)=(m2b3m3b2)2

If m1=0, we can know r=b12 by calculation. So the conclusion is established.

Step 6: If r1,r2 are any two quadratic non-residues, we have

χ(r2r11)=χ(r2)χ(r11)=(1)(1)=1

which means r2r11 is a quadratic residue. So there exists k𝔽q such that k2=r2r11. Consider bijection φ(y)=ky, it maps the element in S(0,r1) to S(0,r2) since for every yS(0,r1), we have

φ(y)2=(ky1)2+(ky2)2+(ky3)2=k2(y12+y22+y32)=k2r1=r2

Therefore we have |S(0,r1)|=|S(0,r2)|.

If r1,r2 are both non-zero quadratic residues, assuming r1=a2 and r2=b2, we can consider bijection φ(y)=ba1y. Then we know |S(0,r1)|=|S(0,r2)| similarly.

For the case of r=0, we consider the equation of y12+y22=y32. Using the formula in [6]:

N(c)=qχ(1) for c0andN(0)=q+χ(1)(q1)

where N(c) represents the number of roots of u2+v2=c in 𝔽q. If y3=0, we have N(0)=2q1. If y30, we have N(y32)=q1. Adding them up and we get |S(0,0)|=2q1+(q1)(q1)=q2.

For the case of r=s20, we consider the equation of y12+y22=s2y32. If y3=±s, we know N(0)=2q1. If y3±s, we know N(s2y32)=q1. Adding them up and we get |S(0,s2)|=2(2q1)+(q2)(q1)=q2+q.

For the case of r is quadratic non-residue, we consider the equation of y12+y22=ry32. For every y3, we have ry320. Since N(ry32)=q1, we know |S(0,r)|=q(q1)=q2q. ∎

Solution 8.3.

Consider the equation of quadratic surface: Q(x)=xAx+bx+c=0, where A is a symmetric matrix and b is a vector. Since the scalar equivalence, we know its degrees of freedom can be realized as a 9-dimension projective space. Every line li can be parameterized as ri(t)=pi+tdi. So Q(pi+tdi)=0 for i=1,2,3 and every t, which means the coefficients of a quadratic polynomial in t are all 0. That is, for i=1,2,3 and every t,

diAdi=0, 2piApi+bdi=0,piApi+bpi+c=0

There are 9 linear equations. Then the dimension of solution is 0, in the sense of scalar equivalence. So there is only one degree 2 algebraic surface containing the three lines. ∎

Solution 8.4.

About the smoothness of R(l1,l2,l3), we consider the singularity p such that Q(p)=0 and Q(p)=2Ap+b=0. Assuming :r(t)=pt+d is a line passing p, since the two conditions above, we have

Q(p+td)=(p+td)A(p+td)+b(p+td)+c=t2(dAd)

Let d=qp, here qR(l1,l2,l3), we know dAd=0, which means the line passing p,q is on R(l1,l2,l3). Since the conclusion above is established for any qR(l1,l2,l3), we know R(l1,l2,l3) is a conical surface with vertex p. Then l1,l2,l3 won’t be skew because they all passes p, which leads a contradiction.

For any point pR(l1,l2,l3), assume that line r(t)=p+td passes it. Using Taylor expansion, we know Q(p+td)=Q(p)+tQ(p)d+t2dAd=t(Q(p)d+)t2(dAd). Now we have Q(p)d=0 and dAd=0. Equation Q(p)d=0 shows d is on tangent plane passing p. Notice that reguli are irreducible, we know dAd is not always 0 in that tangent plane. (Otherwise, any d0 will match the condition above, which means R(l1,l2,l3) vanish in that tangent plane, leading that R(l1,l2,l3) is reducible.) Since dAd is quadratic, it has at most two linear factors. That is, there are at most two lines satisfy the condition above.

For other fields, the conclusions are also true. For 𝔽𝔽2n, the proof above also works.. When 𝔽=𝔽2n, the proof should be modified a bit.

Regarding of the smoothness, if there is a singularity, we know b=0. Choose p,qR(l1,l2,l3), we have pAp+c=0 and qAq+c=0. Let r(t)=p+t(qp), then

Q(r(t))=(p+t(qp))A(p+t(qp))+c

Expand it and use pAq=qAp and 2=0 to simplify, we have Q(r(t))=0, which means r(t)R(l1,l2,l3). Because it is established for any p,q, we know R(l1,l2,l3) contains a plane, which is contradictory to that R(l1,l2,l3) is irreducible.

Regarding that there are at most two lines in R(l1,l2,l3) passing one point, just expand Q(p+td)=(p+td)A(p+td)+b(p+td)+c and simplify. We can get Q(p+td)=t(bd)+t2(dAd), which means bd=0 and dAd=0 if r(t)=p+tdR(l1,l2,l3) for any t. And that’s contradictory to that R(l1,l2,l3) is irreducible. ∎

Solution 8.5.

With a similar proof of Theorem 8.18, we can know:

Claim. Suppose that 𝔏 is a set of L lines in 𝔽3 with B lines in any plane or degree 2 surface. Then the number of intersection points of 𝔏 is KB13L53, where K1 is constant .

Just replace all ”10” of the proof of Theorem 8.18 with ”B”, we can finish proof of the claim above.

We will prove by induction that a set of L lines determines at most CL74 joints. More precisely, here C=K34214.

Take B=K34234L14, let β=K34234, now B=βL14. If L=3, we have J(𝔏)1 and K34214L741. So the inductive foundation is established. We can assume that, for any 𝔏 containing L<L lines, we have J(𝔏)C(L)74. Now consider the line set 𝔏 with L lines.

For the case that there is no plane or regulus containing more than B lines, consider the number of intersections, we have J(𝔏)KB13L53=Kβ13L74=CL74.

If there exist a plane or regulus containing more than B lines, we call it Σ and let 𝔏Σ𝔏 denote the lines in Σ. Notice that |𝔏Σ|>B, let 𝔏=𝔏\𝔏Σ and we have |𝔏|<LB. If Σ is plane, any three lines are coplanar,then there doesn’t exist joint on Σ. If Σ is regulus, any point lies on at most two lines, then there doesn’t exist joint on it. So there’s no joint in 𝔏Σ.

The joints in 𝔏 can be divided into two categories. In the first category, the three lines that make up the joints are all in 𝔏, the number of these joints is J(𝔏). In the second category, those three lines have at least one in Σ and at least one in 𝔏. For this category, every line in 𝔏 intersects Σ at most 2 points. So the number of joints in the second category is at most 2|𝔏|2L. Now we have

J(𝔏)|J(𝔏)|+2LC(LB)74+2LCL74

The last inequality holds when L3 and K1, here we omit some of the calculation details. So the induction finishes and conclusion is established. ∎

Solution 10.1.

Let D be a degree that we will choose later. By Theorem 10.3, we can find a non-zero polynomial P of degree D so that each component of the complement of Z(P) contains SD2 points of 𝒮. Let Oi be the components of 2\Z(P). For each i, let 𝒮i=𝒮Oi and let ΓiΓ be the set of unit circles of Γ that intersect Oi. Let Si=|𝒮i| and Ni=|Γi|. Since circle is a quadratic curve, if it isn’t contained in Z(P), then it intersects Z(P) in at most 2D points, by Bezout Theorem. Thus every unit circle passes D cells, which means iNiND.

Claim. If 𝒮 is a set of S points in 2, and Γ is a set of N unit circles in 2, then
(1) I(𝒮,Γ)N+2S2;
(2) I(𝒮,Γ)2N2+S.

Fix x𝒮. Let Nx be the number of circles of Γ that contain x and no other point of 𝒮. For each other point yS, there is at most two circles of Γ containing x and y. Therefore, I(x,Γ)2S+Nx. So I(𝒮,Γ)2S2+x𝒮Nx2S2+N. The proof of the other inequality is similar.

If N>S2 or S>N2, then the conclusion follows from the claim above. Therefore, we can now restrict to the case that S12NS2.

Applying the claim in each cell, we get I(𝒮i,Γi)Ni+2Si2 then

I(𝒮cell,Γ)=iI(𝒮i,Γi)iNi+iSi2ND+SD2iSiND+S2D2

Since every circle intersects Z(P) in at most 2D points if it isn’t contained in Z(P), we have I(𝒮alg,Γcell)2ND. Consider the degree, the number of circles in Z(P) is at most D2. Using the claim above, we get I(𝒮alg,Γalg)S+D22.

Adding them up and we know I(𝒮,Γ)ND+S2D2+S+D2. Take DS23N13, from NS2, we know S23N131. Also from S12N, we know D2S43N23S. Therefore, I(𝒮,Γ)S23N23+S. And let Γ be the unit circles centered at 𝒮, we know S=N and I(𝒮,Γ)N43, which means a set of N points in the plane determines N43 unit distances. ∎

Solution 10.2.
99 9 Conclusions needed to prove should be I(𝒮,Γ)S3+N and I(𝒮,Γ)S35N45+N+S.

Consider the circumcenter of a triangle, we can know a set of 3 points lies on a unique circle immediately. Fix x𝒮. Let ΓhighΓ be the circles containing at least 3 points in 𝒮 and Γlow=Γ\Γhigh. Therefore, I(x,Γ)=I(x,Γhigh)+I(x,Γlow)(S12)+I(x,Γlow). So I(𝒮,Γ)SS2+x𝒮I(x,Γlow)S3+(N+2N)S3+N.

Let D be a degree that we will choose later. By Theorem 10.3, we can find a non-zero polynomial P of degree D so that each component of the complement of Z(P) contains SD2 points of 𝒮. Let Oi be the components of 2\Z(P). For each i, let 𝒮i=𝒮Oi and let ΓiΓ be the set of circles of Γ that intersect Oi. Let Si=|𝒮i| and Ni=|Γi|. Since circle is a quadratic curve, if it isn’t contained in Z(P), then it intersects Z(P) in at most 2D points, by Bezout Theorem. Thus every circle passes D cells, which means iNiND.

Applying that in each cell, we get

I(𝒮cell,Γ)=iI(𝒮i,Γi)iNi+iSi3ND+(SD2)2iSiND+S3D4

Since every circle intersects Z(P) in at most 2D points if it isn’t contained in Z(P), we have I(𝒮alg,Γcell)2ND. Consider the degree, the number of circles in Z(P) is at most D2.

We can prove I(𝒮,Γ)S+2N2 by the method in Exercise 10.1. Using the inequality above, we get I(𝒮alg,Γalg)S+D22.

If N>S3 or S>N2, then we know I(𝒮,Γ)N or S respectively. Therefore, we can now restrict to the case that S12NS3.

Adding them up and we know I(𝒮,Γ)ND+S3D4+S+D2. Take DS35N15, from NS3, we know S35N131. Also from S12N, we know D2S65N25S. Therefore, I(𝒮,Γ)S35N45+S.

Regarding of the r -rich point. Consider the Pr(Γ) as 𝒮, we have S2(N2)N2. For the case of NS3, we have I(𝒮,Γ)S35N45 and I(𝒮,Γ)rS. Then we have SN2r52 immediately.1010 10 If N>S3, the conclusion may be not established. Consider an counterexample: there is a point lying on N circles and we have PN(Γ)1, but let N we have PN(Γ)N120.

Solution 10.3.

Suppose that Γ is a set of N parabolas in the plane and 𝒮 is a set of S points in the plane. Here is an example of I(𝒮,Γ)S23N23. Take Y=3X2,A=1,B=X,C=X2, we assume X as a large integer. Notice that for every integer x such that |x|X, we have (x,ax2+bx+c)𝒮 since |ax2+bx+c|x2+|bx|+|c|3X2Y. Now we have I(𝒮,Γ)=N2XX4 and SNX3. Thus we have I(𝒮,Γ)S23N23. ∎

Solution 10.4.

First, if there exist two irreducible algebraic curves γ1 and γ2 with degree at most d, the number of intersections of them is at most d2, since Bezout Theorem. So for any set of d2+1 points in 2, there is at most one curve γΓ containing all the points.

Fix x𝒮. Let ΓhighΓ be the irreducible algebraic curves containing at least k points in 𝒮 and Γlow=Γ\Γhigh. Therefore, I(x,Γ)=I(x,Γhigh)+I(x,Γlow)(S1k1)+I(x,Γlow). So I(𝒮,Γ)SSk1+x𝒮I(x,Γlow)Sk+(N+2N++(k1)N)Sk+N.

Let D be a degree that we will choose later. By Theorem 10.3, we can find a non-zero polynomial P of degree D so that each component of the complement of Z(P) contains SD2 points of 𝒮. Let Oi be the components of 2\Z(P). For each i, let 𝒮i=𝒮Oi and let ΓiΓ be the set of irreducible algebraic curves of Γ that intersect Oi. Let Si=|𝒮i| and Ni=|Γi|. Since the degree of irreducible algebraic curves is at most d, if it isn’t contained in Z(P), then it intersects Z(P) in at most dD points, by Bezout Theorem. Thus every irreducible algebraic curve passes D cells, which means iNiND.

Applying that in each cell, we get

I(𝒮cell,Γ) =iI(𝒮i,Γi)iNi+iSik
ND+(SD2)k1iSiND+SkD2(k1)

Since every irreducible algebraic curve intersects Z(P) in at most dD points if it isn’t contained in Z(P), we have I(𝒮alg,Γcell)NdD. Consider the degree, the number of irreducible algebraic curves in Z(P) is at most Dd.

We can prove I(𝒮,Γ)S+d2N2 by the method in Exercise 10.1. Using the inequality above, we get I(𝒮alg,Γalg)S+D2.

If NSk or S>N2, then we know I(𝒮,Γ)N or S respectively. Therefore, we can now restrict to the case that S12NSk.

Adding them up and we know I(𝒮,Γ)ND+SkD2(k1)+S+D2. Take DSk2k1N12k1, from NSk, we know Sk2k1N12k11. Also from S12N, we know D2S2k2k1N22k1S. Therefore, I(𝒮,Γ)Sk2k1N2k22k1+S. ∎

Solution 10.5.

We will use induction to prove the following claim firstly.

Claim. For any ε>0, if 𝔏 is a set of L lines in the plane, and 𝒮 is a set of S points in the plane, then I(𝒮,𝔏)C(ε)S23+εL23+ε+(D2+1)(S+L).

We can observe that the conclusion is established when S=1,L=1. Assuming that the conclusion is true for 𝒮,𝔏 such that |𝒮|<|𝒮|,|𝔏|<|𝔏|.

By Theorem 10.3, we can find a non-zero polynomial P of degree D so that each component of the complement of Z(P) contains KSD2 points of 𝒮, where K is a constant. Let kD2 is the number of cells and Oi be the components of 2\Z(P). For each i, let 𝒮i=𝒮Oi and let 𝔏i𝔏 be the set of lines of 𝔏 that intersect Oi. Let Si=|𝒮i| and Li=|𝔏i|. Since every line intersects Z(P) in at most D points, then every line passes D+1 cells, which means i=1kD2LiL(D+1).

If NS2 or S>N2, then we know I(𝒮,𝔏)2N or 2S respectively. Therefore, we can now restrict to the case that S12NS2.

In following content, we assume that

KpD24p(D+1)pk1p<1andC(ε)(2+D)(D2+1)1KpD24p(D+1)pk1p

which are possible when D=D(ε) and C(ε) are both large, where p=23+ε<1.

Applying that in each cell, using Jensen’s inequality, we get

I(𝒮cell,𝔏) =i=1kD2I(𝒮i,𝔏i)i=1kD2C(ε)((KSiD2)pLip)+(D2+1)(Si+Li)
C(ε)KpD2p(kD2)Sp(L(D+1)kD2)p+(D2+1)(S+L(D+1))
=C(ε)KpD24p(D+1)pk1pSpLp+(D2+1)(2+D)SpLp
C(ε)SpLp

Also, similar to the proof of Theorem 10.10, we know I(𝒮alg,𝔏cell)LD and I(𝒮alg,𝔏alg)S+D2. Then we know

I(𝒮alg,𝔏cell)+I(𝒮alg,𝔏alg)LD+S+D2(D2+1)(S+L)

since S,L1 and D1. Now adding them up and we finish the proof of this claim.

Notice that C(ε)>D2+1, we have

I(𝒮,𝔏)C(ε)S23+εL23+ε+(D2+1)(S+L)C(ε)(S23+εL23+ε+S+L)

Solution 10.6.

Fix x𝒮. Let Nx be the number of planes of Γ that contain x and no other point of 𝒮. For each other point yS, there is at most two planes of Γ containing x and y. Therefore, I(x,Γ)2S+Nx. So I(𝒮,Γ)2S2+x𝒮Nx2S2+N.

Fix γΓ. Let 𝒮high𝒮 be the points contained in least 3 planes in Γ and 𝒮low=𝒮\𝒮high. Therefore, I(𝒮,γ)=I(𝒮high,γ)+I(𝒮low,γ)(N12)+I(𝒮low,γ). So I(𝒮,Γ)NN22+γΓI(𝒮low,γ)N32+(S+2S)N32+3S.

Claim. For any ε>0, if Γ is a set of N planes in 3, and 𝒮 is a set of S points in 3, then I(𝒮,Γ)C(ε)S45+εN35+ε+(D3+3)(S+N).

We can observe that the conclusion is established when S=1,L=1. Assuming that the conclusion is true for 𝒮,𝔏 such that |𝒮|<|𝒮|,|𝔏|<|𝔏|.

By Theorem 10.3, we can find a non-zero polynomial P of degree D so that each component of the complement of Z(P) contains KSD3 points of 𝒮, where K is a constant. Let kD3 is the number of cells and Oi be the components of 3\Z(P). For each i, let 𝒮i=𝒮Oi and let ΓiΓ be the set of planes of Γ that intersect Oi. Let Si=|𝒮i| and Ni=|Γi|. According to Harnack inequality, every plane intersects Z(P) in at most MD2 cells, which means i=1kD3NiMND2, where M is a constant. Assume Γalg is the set of planes of Γ that lie in Z(P) and Γcell=Γ\Γalg.

1111 11 Idea of this part comes from [2].

Let 𝒮alg be the set of points of 𝒮 that lie in Z(P) and 𝒮algalg the set of points of 𝒮alg that lie in Z(Q). Again by Theorem 10.3, we can find a non-zero polynomial Q of degree D which is co-prime to P so that each component of the complement of Z(Q) contains TSD3 points of 𝒮alg, where T is a constant. (That is possible because if not, we can consider a small perturbation of Q to make they are co-prime.) Let tD3 is the number of cells and Ui be the components of 3\Z(Q). For each i, let 𝒮algi=𝒮algUi and let ΓiΓ be the set of planes of Γ that intersect Ui. Let si=|𝒮algi| and ni=|Γi|. According to Harnack inequality, every plane intersects Z(Q) in at most mD2 cells, which means i=1tD3nimND2, where m is a constant.

If N>S2 or S>N3, then we know I(𝒮,𝔏)3N or 4S respectively. Therefore, we can now restrict to the case that S13NS2.

In following content, we assume that

Kpk1pD33pqMq+Tpt1pD33pqmq<1

and

C(ε)(D3+3)(MD2+mD2+2)1Kpk1pD33pqMqTpt1pD33pqmq

which are possible when D=D(ε) and C(ε) are both large, where p=35+ε and q=45+ε<1.

Applying that in each cell, using Jensen’s inequality, let 𝒮cell=𝒮\𝒮alg, we get

I(𝒮cell,Γ) =i=1kD3I(𝒮i,Γi)i=1kD3C(ε)((KSiD3)pNiq)+(D3+3)(Si+Ni)
C(ε)KpD3p(kD3)Sp(MND2kD3)q+(D3+3)(S+MD2N)
=C(ε)Kpk1pD33pqMqSpNq+(D3+3)(1+MD2)SpNq

Assume 𝒮algcell=𝒮alg\𝒮algalg, we have

I(𝒮algcell,Γ) =i=1tD3I(𝒮algi,Γi)i=1tD3C(ε)((TsiD3)pniq)+(D3+3)(si+ni)
C(ε)TpD3p(tD3)Sp(mND2tD3)q+(D3+3)(S+mD2N)
=C(ε)Tpt1pD33pqmqSpNq+(D3+3)(1+mD2)SpNq

Now we know I(𝒮cell,Γ)+I(𝒮algcell,Γ)C(ε)SpNq.

Consider Bezout Theorem, each plane of Γ has at most D2 intersection points with Z(P)Z(Q), and so it has at most D2 incidences with 𝒮alg, we know I(𝒮algalg,Γcell)ND2. And using the inequality above, we have I(𝒮algalg,Γalg)3S+D32. Then we know

I(𝒮algalg,Γcell)+I(𝒮algalg,Γalg)ND2+3S+D32(D3+3)(S+N)

since S,L1 and D1. Now adding them up and we finish the proof of this claim.

Notice that C(ε)>D3+3, we have

I(𝒮,Γ)C(ε)S45+εN35+ε+(D3+3)(S+N)C(ε)(S45+εN35+ε+S+N)

Solution 10.7.

Considering a circle S large enough, which isn’t contained in Z(P), we know |Z(P)S|2D by Bezout Theorem. Notice that every unbounded component must intersect S, we know the number of unbounded components is at most 2D.

Regarding the bounded components, in every bounded component U, we know P must reach a local maximum or a local minimum, which means there is a critical point in U. If there’s no common factor of 1P and 2P, then the number of points such that 1P=2P=0 is at most (deg1P)(deg2P)(D1)2. Then the number of bounded components is at most (D1)2.

For the case of 1P and 2P have at least one common factor, we can consider the polynomial Q=P+w1x1+w2x2. Let m=minU boundedmaxxU|P(x)|, we take w1 and w2 small enough such that |w1x1+w2x2|<m2 for x in every bounded component. So for every bounded U, there exist a point x in it such that |Q(x)|m|w1x1+w2x2|>m2. And for xUZ(P), we know |Q(x)|=|w1x1+w2x2|<m2. Thus there is a critical point of Q in U. Notice that, for c small enough (for all 0<c<K where K is a constant depending P), 1P+c and 2P+c have no common factor, we know that we can always make 1Q and 2Q have no common factor. Therefore we know the number of bounded components is at most (D1)2. The number of components of 2\Z(P) is at most D2+1D2. ∎

Solution 10.8.

Suppose that there are three unit 2-spheres S1,S2,S3. Notice that S1S2 and S1S3 are both circles and S1S2S3=(S1S2)(S1S3), we know |S1S2S3|2 since two circles intersect in at most 2 points.

Fix x𝒮. Let ΓhighΓ be the unit 2-spheres containing at least 3 points in 𝒮 and Γlow=Γ\Γhigh. Notice that 3 points can determine at most two unit 2-spheres. Therefore, I(x,Γ)=I(x,Γhigh)+I(x,Γlow)2(S12)+I(x,Γlow). So I(𝒮,Γ)SS2+x𝒮I(x,Γlow)S3+(N+2N)=S3+3N.

Fix γΓ. Let 𝒮high𝒮 be the points contained in least 3 unit 2-spheres in Γ and 𝒮low=𝒮\𝒮high. Notice that 3 unit 2-spheres intersect in at most two points. Therefore, I(𝒮,γ)=I(𝒮high,γ)+I(𝒮low,γ)2(N12)+I(𝒮low,γ). So I(𝒮,Γ)NN2+γΓI(𝒮low,γ)N3+(S+2S)=N3+3S.

Claim. For any ε>0, if Γ is a set of N unit 2-spheres in 3, and 𝒮 is a set of S points in 3, then I(𝒮,Γ)C(ε)S34+εN34+ε+(2D3+3)(S+N).

By Theorem 10.3, we can find a non-zero polynomial P of degree D so that each component of the complement of Z(P) contains KSD3 points of 𝒮, where K is a constant. Let kD3 is the number of cells and Oi be the components of 3\Z(P). For each i, let 𝒮i=𝒮Oi and let ΓiΓ be the set of unit 2-spheres of Γ that intersect Oi. Let Si=|𝒮i| and Ni=|Γi|. According to spherical version of the Harnack inequality, every unit 2-sphere intersects Z(P) in at most MD2 cells, which means i=1kD3NiMND2, where M is a constant. Assume Γalg is the set of unit 2-spheres of Γ that lie in Z(P) and Γcell=Γ\Γalg.

Let 𝒮alg be the set of points of 𝒮 that lie in Z(P) and 𝒮algalg the set of points of 𝒮alg that lie in Z(Q). Again by Theorem 10.3, we can find a non-zero polynomial Q of degree D which is co-prime to P so that each component of the complement of Z(Q) contains TSD3 points of 𝒮alg, where T is a constant. (That is possible because if not, we can consider a small perturbation of Q to make they are co-prime.) Let tD3 is the number of cells and Ui be the components of 3\Z(Q). For each i, let 𝒮algi=𝒮algUi and let ΓiΓ be the set of unit 2-spheres of Γ that intersect Ui. Let si=|𝒮algi| and ni=|Γi|. According to spherical version of the Harnack inequality, every unit 2-spheres intersects Z(Q) in at most mD2 cells, which means i=1tD3nimND2, where m is a constant.

If N>S3 or S>N3, then we know I(𝒮,𝔏)4N or 4S respectively. Therefore, we can now restrict to the case that S13NS3.

In following content, we assume that

Kpk1pD34pMp+Tpt1pD34pmp<1

and

C(ε)(D3+3)(MD2+mD2+2)1Kpk1pD33pqMqTpt1pD33pqmq

which are possible when D=D(ε) and C(ε) are both large, where p=34+ε<1.

Applying that in each cell, using Jensen’s inequality, let 𝒮cell=𝒮\𝒮alg, we get

I(𝒮cell,Γ) =i=1kD3I(𝒮i,Γi)i=1kD3C(ε)((KSiD3)pNip)+(2D3+3)(Si+Ni)
C(ε)KpD3p(kD3)Sp(MND2kD3)p+(2D3+3)(S+MD2N)
=C(ε)Kpk1pD34pMpSpNp+(2D3+3)(1+MD2)SpNp

Assume 𝒮algcell=𝒮alg\𝒮algalg, we have

I(𝒮algcell,Γ) =i=1tD3I(𝒮algi,Γi)i=1tD3C(ε)((TsiD3)pnip)+(2D3+3)(si+ni)
C(ε)TpD3p(tD3)Sp(mND2tD3)p+(2D3+3)(S+mD2N)
=C(ε)Tpt1pD34pmpSpNp+(2D3+3)(1+mD2)SpNp
C(ε)SpNp

Now we know I(𝒮cell,Γ)+I(𝒮algcell,Γ)C(ε)SpNq.

Consider Bezout Theorem and that sphere is irreducible in , each unit 2-sphere of Γ has at most 2D2 intersection points with Z(P)Z(Q), and so it has at most 2D2 incidences with 𝒮alg, we know I(𝒮algalg,Γcell)2ND2. Consider the degree, the number of unit 2-sphere in Z(P) is at most D2. And using the inequality above, we have I(𝒮algalg,Γalg)3S+(D2)3. Then we know

I(𝒮algalg,Γcell)+I(𝒮algalg,Γalg)2ND2+3S+(D2)3(2D3+3)(S+N)

since S,L1 and D1. Now adding them up and we finish the proof of this claim.

Notice that C(ε)>2D3+3, we have

I(𝒮,Γ)C(ε)S34+εN34+ε+(D3+3)(S+N)C(ε)(S34+εN34+ε+S+N)

Take Γ as the unit 2-spheres centered at points in 𝒮 and we have N=S. And we can get that N points in 3 determine at most N32+ε unit distances immediately. ∎

Solution 10.9.

Fix x𝒮. Let ΓhighΓ be the 2-spheres containing at least 11 points in 𝒮 and Γlow=Γ\Γhigh. Notice that 11 points can determine a sphere. Therefore, I(x,Γ)=I(x,Γhigh)+I(x,Γlow)(S110)+I(x,Γlow). So I(𝒮,Γ)SS2+x𝒮I(x,Γlow)S11+(N+2N++10N)=S11+55N.

Fix γΓ. Let Sγ be the number of points of 𝒮 that contained in γ and no other sphere of Γ. Notice that two spheres intersect in at most 10 points. Therefore, I(𝒮,γ)Sγ+10N, so we have I(𝒮,Γ)N(10N)+γΓSγ10N2+S10N3+S. 1212 12 Since here we can use a more accurate estimate of 10N2+S, it is possible that the result of this problem can be better.

Claim. For any ε>0, if Γ is a set of N 2-spheres in 3, and 𝒮 is a set of S points in 3, then I(𝒮,Γ)C(ε)S1116+εN1516+ε+(2D3+1)(S+N).

By Theorem 10.3, we can find a non-zero polynomial P of degree D so that each component of the complement of Z(P) contains KSD3 points of 𝒮, where K is a constant. Let kD3 is the number of cells and Oi be the components of 3\Z(P). For each i, let 𝒮i=𝒮Oi and let ΓiΓ be the set of 2-spheres of Γ that intersect Oi. Let Si=|𝒮i| and Ni=|Γi|. According to spherical version of the Harnack inequality, every 2-sphere intersects Z(P) in at most MD2 cells, which means i=1kD3NiMND2, where M is a constant. Assume Γalg is the set of 2-spheres of Γ that lie in Z(P) and Γcell=Γ\Γalg.

Let 𝒮alg be the set of points of 𝒮 that lie in Z(P) and 𝒮algalg the set of points of 𝒮alg that lie in Z(Q). Again by Theorem 10.3, we can find a non-zero polynomial Q of degree D which is co-prime to P so that each component of the complement of Z(Q) contains TSD3 points of 𝒮alg, where T is a constant. (That is possible because if not, we can consider a small perturbation of Q to make they are co-prime.) Let tD3 is the number of cells and Ui be the components of 3\Z(Q). For each i, let 𝒮algi=𝒮algUi and let ΓiΓ be the set of 2-spheres of Γ that intersect Ui. Let si=|𝒮algi| and ni=|Γi|. According to spherical version of the Harnack inequality, every 2-spheres intersects Z(Q) in at most mD2 cells, which means i=1tD3nimND2, where m is a constant.

If N>S11 or S>N3, then we know I(𝒮,𝔏)56N or 11S respectively. Therefore, we can now restrict to the case that S13NS11.

In following content, we assume that

Kpk1pD33pqMq+Tpt1pD33pqmq<1

and

C(ε)(2D3+1)(MD2+mD2+2)1Kpk1pD33pqMqTpt1pD33pqmq

which are possible when D=D(ε) and C(ε) are both large, where p=1116+ε and q=1516+ε<1.

Applying that in each cell, using Jensen’s inequality, let 𝒮cell=𝒮\𝒮alg, we get

I(𝒮cell,Γ) =i=1kD3I(𝒮i,Γi)i=1kD3C(ε)((KSiD3)pNiq)+(2D3+1)(Si+Ni)
C(ε)KpD3p(kD3)Sp(MND2kD3)q+(2D3+1)(S+MD2N)
=C(ε)Kpk1pD33pqMqSpNq+(2D3+1)(1+MD2)SpNq

Assume 𝒮algcell=𝒮alg\𝒮algalg, we have

I(𝒮algcell,Γ) =i=1tD3I(𝒮algi,Γi)i=1tD3C(ε)((TsiD3)pniq)+(2D3+1)(si+ni)
C(ε)TpD3p(tD3)Sp(mND2tD3)q+(2D3+1)(S+mD2N)
=C(ε)Tpt1pD33pqmqSpNq+(2D3+1)(1+mD2)SpNq

Now we know I(𝒮cell,Γ)+I(𝒮algcell,Γ)C(ε)SpNq.

Consider Bezout Theorem and that sphere is irreducible in , each 2-sphere of Γ has at most 2D2 intersection points with Z(P)Z(Q), and so it has at most 2D2 incidences with 𝒮alg, we know I(𝒮algalg,Γcell)2ND2. Consider the degree, the number of unit 2-sphere in Z(P) is at most D2. And using the inequality above, we have I(𝒮algalg,Γalg)S+10(D2)3S+2D3. Then we know

I(𝒮algalg,Γcell)+I(𝒮algalg,Γalg)2ND2+S+2D3(2D3+1)(S+N)

since S,L1 and D1. Now adding them up and we finish the proof of this claim.

Notice that C(ε)>2D3+1, we have

I(𝒮,Γ)C(ε)S1116+εN1516+ε+(2D3+1)(S+N)C(ε)(S1116+εN1516+ε+S+N)

Solution 10.10.

By Theorem 10.3, there exist a polynomial PPolyD(3) such that 3\Z(P) is decomposed into D3 cells, satisfying |OiPr(𝔏)|D3|Pr(𝔏)| for every i. We have |Pr(𝔏)|=|Pr(𝔏)Z(P)|+|Pr(𝔏)(3)\Z(P)|.

For the case of |Pr(𝔏)Z(P)||Pr(𝔏)(3)\Z(P)|, we know for every Oi, we have |Pr(𝔏)|D3|Pr(𝔏)Oi|D3|Pr(𝔏i)|, where 𝔏i𝔏 is set of lines that intersect Oi. Notice that every line intersects at most D+1 cells, we have i|𝔏i|(D+1)LDL. Since the number of cells is D3, there exist a cell Oi such that |𝔏i|LD2. So there is

|Pr(𝔏)|D3|Pr(𝔏i)|D3I(B,D,|𝔏i|,r)D3I(B,D,KLD2,r)

For the case of |Pr(𝔏)Z(P)||Pr(𝔏)(3)\Z(P)|, we know |Pr(𝔏)|2|Pr(𝔏)Z(P)|DLr1+B2r3 by Lemma 10.15.

Therefore, we know I(B,D,L,r)(D3I(B,D,KLD2,r)+DLr1+B2r3), which means I(B,D,L,r)M(D3I(B,D,KLD2,r)+DLr1+B2r3). Take C=max{M,K}, we get

I(B,D,L,r)C(D3I(B,D,CLD2,r)+DLr1+B2r3)

Solution 10.11.

We will use induction on L to prove this conclusion. It obvious that it is established for L=1. We know there is a constant K such that I(B,D,L,r)K(D3I(B,D,KLD2,r)+DLr1+B2r3). We assume that this conclusion is established for 𝔏 with |𝔏|=LL, that is, I(B,D,L,r)C(ε)B12ε(L)32+εr2.

Suppose that C(ε) and D=D(ε) large enough, satisfying the following four conditions:

K52+εD2ε13,KD212,KD16C(ε),KB32+ε13C(ε)

Since KLD2L2, we can use the induction hypothesis that

I(B,D,KLD2,r)C(ε)B12ε(KLD2)32+εr2

So we have

I(B,D,L,r)K(D3C(ε)B12ε(KLD2)32+εr2+DLr1+B2r3)

that is

I(B,D,L,r)C(ε)K52+εD2εB12εL32+εr2+KDLr1+KB2r3

Use the conditions above, we have

C(ε)K52+εD2εB12εL32+εr213C(ε)B(1/2)εL(3/2)+εr2
KDLr113C(ε)B(1/2)εL(3/2)+εr2
KB2r313C(ε)B(1/2)εL(3/2)+εr2

Thus we have I(B,D,L,r)C(ε)B12εL32+εr2, which leads the conclusion. ∎

Solution 10.12.

For the case of r>2L12, then Pr(𝔏)2Lr1 by Lemma 10.16.

If r2L12, notice that Lemma 10.15 is also available for n and also sharp, because all B lines can be on the same 2-plane contained by Z(P). Similar to Exercise 10.10, we can prove that there exists a constant K such that

I(B,D,L,r)K(DnI(B,D,CLD(n1),r)+DLr1+B2r3)

Claim. We have I(B,D,L,r)C(ε)Bn2n1εLnn1+εrn+1n1.

We use induction on L to prove this conclusion. It obvious that it is established for L=1. We assume that this conclusion is established for 𝔏 with |𝔏|=LL, that is, I(B,D,L,r)C(ε)Bn2n1ε(L)nn1+εrn+1n1.

Suppose that C(ε) and D=D(ε) large enough, satisfying the following four conditions:

K2n1n1+εD(n1)ε13,KD(n1)12,KD1322n1C(ε),KBnn1+ε13C(ε)

Since KLD(n1)L2, we can use the induction hypothesis that

I(B,D,KLD(n1),r)C(ε)Bn2n1ε(KLD(n1))nn1+εrn+1n1

So we have

I(B,D,L,r)K(DnC(ε)Bn2n1ε(KLD(n1))nn1+εrn+1n1+DLr1+B2r3)

that is

I(B,D,L,r)C(ε)K2n1n1+εD(n1)εBn2n1εLnn1+εrn+1n1+KDLr1+KB2r3

Use the conditions above, we have

K2n1n1+εD(n1)εBn2n1εLnn1+εrn+1n1C(ε)Bn2n1εLnn1+εrn+1n1
KDLr1C(ε)Bn2n1εLnn1+εrn+1n1
KB2r3C(ε)Bn2n1εLnn1+εrn+1n1

Thus we have I(B,D,L,r)C(ε)Bn2n1εLnn1+εrn+1n1. So we have |Pr(𝔏)|C(ε)Bn2n1εLnn1+εrn+1n1. ∎

References

  • [1] Angelo Degree of image of a polynomial map. Note: MathOverflowURL:https://mathoverflow.net/q/63463 (version: 2011-04-29) External Links: https://mathoverflow.net/q/63463, Link Cited by: Solution 6.2.
  • [2] A. Basit and A. Sheffer (2014-Dec.) Incidences with k-non-degenerate sets and their applications. Journal of Computational Geometry 5 (1), pp. 284–302. External Links: Link, Document Cited by: footnote 11.
  • [3] N. Eldredge The lebesgue measure of zero set of a polynomial function is zero. Note: Mathematics Stack ExchangeURL:https://math.stackexchange.com/q/1920527 (version: 2021-07-22) External Links: https://math.stackexchange.com/q/1920527, Link Cited by: footnote 1.
  • [4] R. Hartshorne (1977) Varieties. In Algebraic Geometry, pp. 1–59. External Links: ISBN 978-1-4757-3849-0, Document, Link Cited by: Solution 6.1, Solution 6.1.
  • [5] M. Kohan C1 And C2 are non-crossing if they are translates of a convex disc. Note: Mathematics Stack ExchangeURL:https://math.stackexchange.com/q/3225611 (version: 2019-05-14) External Links: https://math.stackexchange.com/q/3225611, Link Cited by: footnote 5.
  • [6] R. Lidl and H. Niederreiter (1996) Equations over finite fields. In Finite Fields, Encyclopedia of Mathematics and its Applications, pp. 268–346. Cited by: Solution 3.2, Solution 3.2, Solution 8.2.
  • [7] G. Martin Polynomial that can be represented as a linear combination. Note: Mathematics Stack ExchangeURL:https://math.stackexchange.com/q/5051406 (version: 2025-03-30) External Links: https://math.stackexchange.com/q/5051406, Link Cited by: footnote 2.
  • [8] Z. Nie and A. Y. Wang (2015) Hilbert functions and the finite degree zariski closure in finite field combinatorial geometry. Journal of Combinatorial Theory, Series A 134, pp. 196–220. External Links: ISSN 0097-3165, Document, Link Cited by: Solution 2.9.
  • [9] L. A. Szekely (1997) Crossing numbers and hard erdos problems in discrete geometry. Combinatorics, Probability and Computing 6 (3), pp. 353–358. External Links: Document Cited by: footnote 6, footnote 7.